9
$\begingroup$

Find all continuous and bounded functions $g$ with : $$\forall x \in \mathbb R, 4g(x)=g(x+1)+g(x-1)+g(x+\pi)+g(x-\pi).$$

I have posted this question here, but received no answer.

$\endgroup$
7
  • 5
    $\begingroup$ MO is the wrong place to repost unanswered questions of this nature $\endgroup$
    – FShrike
    Feb 5, 2023 at 12:28
  • $\begingroup$ I have try in others forums, but no body have an idea $\endgroup$
    – Dattier
    Feb 5, 2023 at 12:31
  • 11
    $\begingroup$ "Very difficult" phrase makes me smile and quitting reading the rest of the text right away. $\endgroup$
    – Wlod AA
    Feb 5, 2023 at 12:33
  • 1
    $\begingroup$ If view it in a two dimension space, then the structure is discrete harmonic function, but it is bounded so it is constant. $\endgroup$
    – katago
    Feb 5, 2023 at 12:34
  • 1
    $\begingroup$ @WlodAA, re, I agree; such editorialising does not belong in MO titles. I edited it out. $\endgroup$
    – LSpice
    Feb 6, 2023 at 15:06

4 Answers 4

18
$\begingroup$

$\newcommand\de\delta$Considering $g$ a distribution (in the generalized-function sense), let $\hat g$ be the Fourier transform of $g$. Then your functional equation yields $$4\hat g(t)=e^{it}\hat g(t)+e^{-it}\hat g(t)+e^{i\pi t}\hat g(t)+e^{-i\pi t}\hat g(t),$$ or $$(\cos t+\cos\pi t-2)\hat g(t)=0,$$ for real $t$.

The equality $\cos t+\cos\pi t-2=0$ for real $t$ implies $\cos t=1=\cos\pi t$ and hence $t=0$ (because $\pi$ is irrational). So, the support of $\hat g$ is $\{0\}$. So (see e.g. "For every compact subset $K\subseteq U$ there exist constants $C_{K}>0$ and $N_{K}\in \mathbb {N}$ such that for all $f\in C_{c}^{\infty }(U)$ with support contained in $K$ [...]" here), we have $\hat g=\sum_{j=0}^n c_j\de^{(j)}$ for some $n\in\{0,1,\dots\}$ and some complex $c_j$'s, where $\de^{(j)}$ is the $j$th derivative of the delta function. So, $g$ is a polynomial. Since $g$ is bounded, it is constant. $\quad\Box$

$\endgroup$
3
  • $\begingroup$ This idea also looks like it can be used to prove the mean value formula of harmonic function. $\endgroup$
    – katago
    Feb 5, 2023 at 14:13
  • $\begingroup$ @katago : Sorry, I don't what "the mean value formula of harmonic function" is. $\endgroup$ Feb 5, 2023 at 14:19
  • $\begingroup$ Sorry, I mean, using your method seems we can prove $u$ satisfied the mean value property $\Longrightarrow$ $u$ is a harmonic function. If $$ u(x)=\frac{1}{\left|\partial B_r\right|} \int_{\partial B_r(x)} u d S=\frac{1}{\left|B_r\right|} \int_{B_r(x)} u(y) d y $$, let $\hat{u}$ be the Fourier transform of $u$, then $\hat{u}(x)=\frac{1}{\left|\partial B_r\right|} \int_{\partial B_r(x)} \hat{u}(x)e^{iv} d S=\frac{1}{\left|B_r\right|} \int_{B_r(x)} \hat{u}(x)e^{iv} d y$, then $\hat u(x)(1-\int_{\partial B_r(x)}vdS)=0$. but I do not know how to move on. $\endgroup$
    – katago
    Feb 5, 2023 at 16:00
5
$\begingroup$

(Compare On equation $f(z+1)-f(z)=f'(z)$.)

Plug $g(x)=e^{\lambda x}$. We obtain the characteristic equation: $$4=2\cosh \lambda+2\cosh\pi\lambda.$$ This has one real root $\lambda=0$ of multiplicity $2$. This root gives a solution $g(x)=ax+b$ with arbitrary constants $a$ and $b$. Since you are looking for bounded solutions, one has to set $a=0$ and we recover the constant solution of @katago. However there are infinitely many others: Let $\lambda=p+iq$ be any complex solution (there are infinitely many of them). Then any linear combination $$g(x)=\sum c_ke^{\lambda_k x}$$ gives you a solution. In general, this solution is complex, but if you want a real solution, notice that $\lambda_k$ come in complex conjugate pairs, so you have an infinte-dimensional space of real solutions. Now solutions bounded on the real line will correspond to pure imaginary $\lambda$, but the characteristic equation does not have pure imaginary roots.

So every bounded solution is indeed constant. That all solutions are linear combinations of exponentials or their limits is a general theorem cited in the reference that I gave in the beginning.

$\endgroup$
4
$\begingroup$

It seems $g$ can only be a constant function. First, $g$ is constant in any shift of set $f(A_x)=\{x+a+b\pi| a,b\in \mathbb N\}$ by Liouville type theorem for discrete harmonic function, and then we need suitable choose the constant to make the function continuous on $\mathbb R/A_x$. but by Dirichlet approximation theorem we can get $g$ is constant on $\mathbb R$.

$\endgroup$
1
  • $\begingroup$ If change the condition to sublinear instead of bounded then, the function may not be constant. $\endgroup$
    – katago
    Feb 5, 2023 at 13:22
4
$\begingroup$

Iosef's answer shows the only solutions are constants.

Plug …

Edgar, G. A.; Rosenblatt, J. M., Difference equations over locally compact Abelian groups, Trans. Am. Math. Soc. 253, 273-289 (1979). ZBL0417.43006.

At the end, we construct a nontrivial, continuous, bounded, almost-periodic function $F : \mathbb R \to \mathbb C$ satisfying $$ 0 = F(x+1)- F(x-1) -\sqrt{2}F(x+2\pi)+\sqrt{2}F(x-2\pi) . $$

$\endgroup$

Your Answer

By clicking “Post Your Answer”, you agree to our terms of service and acknowledge you have read our privacy policy.

Not the answer you're looking for? Browse other questions tagged or ask your own question.